0 Daumen
752 Aufrufe

Aufgabe:

Gegeben sei die Rekursionsgleichung

\( y_{k+1}=y_{k}+5 \text { mit } y_{1}=4, k \in \mathbb{N} \)

a) Bestimmen Sie \( y_{2}, y_{3}, y_{4}, y_{5} \) und \( y_{6} \).

b) Bestimmen Sie eine explizite (nicht rekursive) Darstellung und beweisen Sie diese.


Mein Ansatz:

a) \( y_{k+1}=y_{1}+5 \quad y_{1}=4 \)
\( y_{1+1}=y_{2}=4+5=9 \)
\( y_{3}=9+5=14 \)
\( k=2 \)
\( y_{4}=14+5=19 \)
\( y_{5}=19+5=24 \)
\( y_{6}=24+5=29 \)
\( y_{k}=5 k-1 \)
\( y k=5=2 y_{5}=5 \cdot 5-1=24 \)

Bei der b.) bin ich mir nicht ganz sicher, da dort für k=5 24 rauskommt und oben für k=5 29. y5 stimmt aber überein. Ansonsten hätte ich noch die Idee: yk+1 = 5*(k+1) -1.

Den Beweis haben wir immer über die vollständige Induktion gemacht. Die habe ich allerdings nie ganz verstanden. Wie müsste ich da dann vorgehen?

Avatar von

1 Antwort

0 Daumen
 
Beste Antwort
Das sieht richtig aus y5 = 24 und y6 ist 29. Das hast du auch oben so stehen

yk = 5·k - 1

Induktionsansatz n = 1

y1 = 5·1 - 1 = 4 --> stimmt

Induktionsschritt n --> n + 1

Wir zeigen das die Formel für n+1 gilt, wenn sie für n gilt.

y(k+1) = 5·(k + 1) - 1 = 5·k + 5 - 1 = 5·k - 1 + 5 = yk + 5 --> stimmt auch.
Avatar von 479 k 🚀
Danke für die Antwort.

Den letzen Schritt verstehe ich aber nicht ganz:

Warum ist 5·k - 1 + 5 = yk + 5?
5k - 1 = yk

Das war unser Induktionsansatz den wir für n voraussetzen um es für n + 1 zu zeigen.

Ein anderes Problem?

Stell deine Frage

Willkommen bei der Mathelounge! Stell deine Frage einfach und kostenlos

x
Made by a lovely community